Q9

 
kwjustin89
Thanks Received: 0
Vinny Gambini
Vinny Gambini
 
Posts: 4
Joined: January 08th, 2011
 
 
 

Q9

by kwjustin89 Tue Jun 21, 2011 12:48 pm

I'm not sure I'm understanding something in the rules or diagram properly. For 9, I seem to be able to find that more than answer choice C could work, which is obviously wrong. Can someone guide me through this question? Which frame applies to this question? Thanks a lot.
 
tianfeng102
Thanks Received: 11
Forum Guests
 
Posts: 21
Joined: August 23rd, 2010
 
This post thanked 1 time.
 
 

Re: Q9

by tianfeng102 Fri Jun 24, 2011 9:37 pm

All three frames apply.
(NOVEL) [play]
(FFF) [f, r] (No more than 4 French works)
(FFF R) [f/r]
(FFF R) [f, r] (No more than 4 French works)

Only C) fits.

Another way to pick the right answer is simply to apply rules to the answer choices:
A) one Russian novel; 5 or 6 works, not 4.
B) two French plays; no more than four French works.
D) one Russian novel, two French plays; no more than four French works.
E) two Russian novels, one French play; no more than four novels.
LSAT could change from demon to darling, if you tame the beast (PrepTest) one after another in 60 days.
User avatar
 
ManhattanPrepLSAT1
Thanks Received: 1909
Atticus Finch
Atticus Finch
 
Posts: 2851
Joined: October 07th, 2009
 
 
 

Re: Q9

by ManhattanPrepLSAT1 Mon Jun 27, 2011 3:30 pm

tianfeng102 Wrote:Another way to pick the right answer is simply to apply rules to the answer choices:
A) one Russian novel; 5 or 6 works, not 4.
B) two French plays; no more than four French works.
D) one Russian novel, two French plays; no more than four French works.
E) two Russian novels, one French play; no more than four novels.


Nice work! There's almost always more than one approach that works, and the more practice we have at seeing things from a different perspective, the more likely we are to get to the solution during the middle of the test when our first approach doesn't work.